You are on page 1of 25

PHY4604 Fall 2006 Final Exam Solutions

PHY 4604 Final Exam Solutions


Wednesday December 6, 2006 (Total Points = 100)
Problem 1 (25 points): Consider the following one-dimensional potential: V = +infinity

0 x<L
 V(x)
V ( x) = − V0 0 ≤ x < L
 + ∞ x≥L
x
Where V0 is a positive (real) constant. Region 2 Region 3
Region 1
(A) Suppose that particles with mass m and energy E > 0 enter from the left in
-V
h2 2
0

region 1 and travel to the right and encounter the potential V(x). Let E = ε L
2mL2
V
and r = 1 + 0 .
E
(1) (5 points) Calculate the probability that particles entering from the left in region 1 will be
reflected back (i.e. calculate the ratio of the probability flux
s r in region 1 traveling to the left to
the incident flux in region 1 traveling to the right, PR = j1 / j1 ). Express your answer in terms of
ε and r.
Answer: PR = 1.
Solution: We look for solutions of the time-independent Schrödinger equation
h 2 d 2ψ ( x ) d 2ψ ( x ) 2m
− 2
+ V ( x )ψ ( x ) = Eψ ( x ) or 2
= − 2 ( E − V ( x ))ψ ( x )
2m dx dx h
with Ψ ( x, t ) = ψ ( x)e − iEt / h . In the region x < 0 (region 1) for E > 0 and V(x) = 0 we have
d 2ψ ( x ) 2mE 2mE ε h 2k 2 h2 2
2
= − 2 ψ ( x ) = − k 2ψ ( x ) with k = = and E = = ε
dx h h2 L 2m 2mL2
The most general solution is
ψ 1 ( x ) = Ae + ikx + Be − ikx ,
In the region 0 < x < L (region 2) we have
ψ 2 ( x ) = Ce + iqx + De − iqx ,
2m( E + V0 ) q V
with q = 2
= rk , where r = = 1 + 0 . In the region x > L (region 3) we have
h k E
ψ 3 ( x) = 0 ,
The boundary conditions at x = L implies that
ψ 2 ( L ) = 0 and hence Ce + iqL + De − iqL = 0 or D = −Ce +2 iqL .
Thus,
ψ 2 ( x ) = C (e + iqx − e +2iqL e − iqx ) ,
At x = 0 the wave function must be continuous with a continuous derivative which yields

Department of Physics Page 1 of 25


PHY4604 Fall 2006 Final Exam Solutions

ψ 1 (0) = ψ 2 (0) and A + B = C (1 − e +2iqL )


dψ 1 ( x ) dψ 2 ( x ) +2iqL
= and ikA − ikB = Ciq (1 + e )
dx x =0 dx x=0
Thus,
A + B = C (1 − e +2iqL ) and A − B = Cr (1 + e +2iqL )
And
2 A = C (( r + 1) + ( r − 1)e +2iqL ) and 2 B = −C (( r − 1) + ( r + 1)e +2iqL )
Thus,
2A ( r − 1) + ( r + 1)e +2 iqL
C= and B = − A.
( r + 1) + ( r − 1)e + 2 iqL ( r + 1) + ( r − 1)e + 2 iqL
The relfection probability is given by
2
hk
| B |2 ( r − 1) + ( r + 1)e +2 iqL  ( r − 1) + ( r + 1)e +2 iqL  ( r − 1) + ( r + 1)e −2 iqL 
PR = m
= =  
+ 2 iqL 

hk
m
| A |2 ( r + 1) + ( r − 1)e + 2 iqL  ( r + 1) + ( r − 1)e  ( r + 1) + ( r − 1)e
− 2 iqL

( r − 1) 2 + ( r + 1) 2 + ( r + 1)( r − 1)e + 2 iqL + ( r + 1)( r − 1)e − 2 iqL
= =1
( r + 1) 2 + ( r − 1) 2 + ( r − 1)( r + 1)e + 2 iqL + ( r − 1)( r + 1)e − 2 iqL

(2) (5 points) Calculate the probability that particles traveling to the right in region 2 will be
reflected back (i.e. calculate the ratio of the probability flux in region 2 traveling to the left to
s r
flux in region 2 traveling to the right, R = j2 / j2 ). Express your answer in terms of ε and r.
Answer: R = 1.
Solution: We see that
hq
| D |2 + 2 iqL 2
R= m
hq
= e = 1.
m
| C |2

(3) (5 points) Calculate the ratio of the probability flux


r rin region 2 traveling to the right to the
incident flux in region 1 traveling to the right, P = j2 / j1 ). Express your answer in terms of ε
and r. What are the maximum and minimum values of P(ε) (express your answer in terms of r)?
Evaluate the maximum and minimum values of P(ε) for the case V0 = 3E. Explain what is going
on.
2r 1 1
Answer: P = , Pmin = r→ , P = r r→ 2
r + 1 + ( r − 1) cos( 2 rε )
2 2
r =2
2 max =2

Solution: We see that

Department of Physics Page 2 of 25


PHY4604 Fall 2006 Final Exam Solutions

hq 2
| C |2 2
P= m
=r =
hk
m
| A| 2
( r + 1) + ( r − 1)e + 2 iqL
4r
=
( r + 1) + ( r − 1) + ( r − 1)( r + 1)e + 2 iqL + ( r − 1)( r + 1)e − 2 iqL
2 2

4r 2r
= =
( r + 1) 2 + ( r − 1) 2 + ( r − 1)( r + 1) 2 cos( 2 qL ) r 2 + 1 + ( r 2 − 1) cos( 2 qL )
2r
= 2
r + 1 + ( r − 1) cos( 2 rε )
2

The minimum value of P ocurs when cos(2rε) = 1 and hence


2r 1 1
Pmin = = r
=2
→ ,
r + 1 + ( r − 1) r
2 2
2
where I used
V0
r = 1+
→ 2 .
E V0 = 3 E
The maximum value of P ocurs when cos(2rε) = -1 and hence
2r
Pmax = = r r→
=2
2.
r + 1 − ( r 2 − 1)
2

h2 2
(B) Now consider the case E < 0 and look for possible bound states. Let E = − ε and
2mL2
h2
V0 = 2
α2.
2mL
(1) (5 points) How many bound states are there if α = π/4? What is the ground state energy (i.e.
what is ε for the ground state)?
Answer: No bound states for α < π/2.
Solution: We look for solutions of the time-independent Schrödinger equation
h 2 d 2ψ ( x ) d 2ψ ( x ) 2m
− 2
+ V ( x )ψ ( x ) = E ψ ( x ) or 2
= − 2 ( E − V ( x ))ψ ( x )
2m dx dx h
with Ψ ( x, t ) = ψ ( x)e − iEt / h
. In the region x < 0 (region 1) for E < 0 and V(x) = 0 we have
d 2ψ ( x ) 2mE − 2mE ε h 2κ 2 h2
= − ψ ( x ) = κ 2
ψ ( x ) with κ = = and E = − = − ε2
dx 2 h2 h 2
L 2 m 2mL 2

The most general solution is


ψ 1 ( x ) = Ae +κx + Be −κx ,
− κx
However, e  → ∞ and hence we must set B = 0. In the region 0 < x < L (region 2) we
x = −∞
have
ψ 2 ( x ) = Ce + iqx + De − iqx ,

Department of Physics Page 3 of 25


PHY4604 Fall 2006 Final Exam Solutions

2m( E + V0 ) 2mV0 2mL2V0


with q = , and q 2
+ κ 2
= . Also, ( qL ) 2
+ (κL ) 2
= = α 2 and
h2 h2 h2
(qL) 2 + ε 2 = α 2 . In the region x > L (region 3) we have
ψ 3 ( x) = 0 ,
The boundary conditions at x = L implies that
ψ 2 ( L ) = 0 and hence Ce + iqL + De − iqL = 0 or D = −Ce +2 iqL .
Thus,
ψ 2 ( x ) = C (e + iqx − e +2iqL e − iqx ) ,
At x = 0 the wave function must be continuous with a continuous derivative which yields
ψ 1 (0) = ψ 2 (0) and A = C (1 − e +2iqL )
dψ 1 ( x ) dψ 2 ( x ) κA = Ciq (1 + e +2 iqL )
= and
dx x=0 dx x =0
Dividing the two equations yields
(1 + e +2 iqL ) (e − iqL + e + iqL )
κ = iq = κ = iq − iqL = − q cot( qL )
(1 − e + 2 iqL ) (e − e + iqL )
If we let y = qL then we see that
κ κL α 2 − y2
− cot( y ) = = = ,
q qL y
h2 2 h2
where y 2 + ε 2 = α 2 and E = − 2
ε = − 2
(α 2 − y 2 ) . The evengy levels are determined by
2mL 2mL
α 2 − y2
letting f ( y ) = − cot( y ) and g ( y ) = and looking for the places where f(y) = g(y).
y
π/2 π 3π/2 2π
5

4
α = π/4
3
g(y) f(y)
2

0
0.0 1.6 3.1 4.7 6.3 7.9 y 9.4

-1

We see that there are no solution for y > α. Thus, there are no bound states if α < π/2.

(2) (5 points) How many bound states are there if α = 11π/4? What is the ground state energy
(i.e. what is ε for the ground state)?

Department of Physics Page 4 of 25


PHY4604 Fall 2006 Final Exam Solutions

h2 2
Solution: There are three bound states. E0 = − ε 0 with ε0 ≈ 8.17.
2mL2
Solution: For α = 11π/4 we have three bound states. The ground state has the smallest y, which
is y ≈ 2.81 and hence
h2 h2 h2 h2
E0 = − (α 2
− y 2
min ) ≈ − ((11π / 4) 2
− ( 2.81) 2
) ≈ − (8.17 ) 2
= − 66.74
2mL2 2mL2 2mL2 2mL2
π/2 π 3π/2 2π
5

4
α = 11π/4
3
g(y) f(y)
2

0
0.0 1.6 3.1 4.7 6.3 7.9 y 9.4

-1

r
Problem 2 (25 points): The Hamiltonian of a charged particle with intrinsic spin S at rest in a
r r r rq
magnetic field is H = − µ ⋅ B = −γS ⋅ B , where γ = (q is the electric charge, m is the mass).
m
r
For spin 1 we have S = S x xˆ + S y yˆ + S z zˆ with
0 1 0 0 −1 0  1 0 0 
h   ih    
Sx = 1 0 1 Sy =  1 0 − 1 S z = h 0 0 0 
2  2
0 1 0 0 1 0   0 0 − 1
 
(A) (2 points) Show that
1 0 0
 
S 2 = S x2 + S y2 + S z2 = 2h 2  0 1 0  .
0 0 1
 
Solution: We see that
 0 1 0  0 1 0 1 0 1
h2    h2  
S =  1 0 1  1
2
x 0 1 = 0 2 0
2   2 
 0 1 0  0 1 0  1 0 1

 0 − 1 0  0 −1 0   1 0 − 1
− h2    h2  
Sy =
2
 1 0 − 1 1 0 − 1 =  0 2 0 
2   2 
 0 1 0  0 1 0  
 −1 0 1 
 1 0 0  1 0 0  1 0 0
    
S z = h  0 0 0  0 0 0  = h  0 0 0 
2 2 2

 0 0 − 1 0 0 − 1 0 0 1
    
Hence,

Department of Physics Page 5 of 25


PHY4604 Fall 2006 Final Exam Solutions

1 0 0
 
S = S + S + S = 2h  0 1 0  .
2 2
x
2
y
2
z
2

0 0 1
 
(B) (6 points) The W+ boson is a spin 1 elementary particle with charge q = +e and mass MW.
r
Suppose a W+ is at rest in a uniform magnetic field which points in the z-direction, B = B0 zˆ .
What are the energy levels and the corresponding eigenkets of the system? What is the ground
state energy and ground state eigenket? Express your answers in terms of ω = γB0 .
1 
 
Answer: The ground state (i.e. lowest) energy is E0 = −hω with eigenket | χ 0 >=| 11 >=  0  .
 0
 
 0
st  
The 1 excited state has energy E1 = 0 with eigenket | χ1 >=| 10 >= 1  .
 0
 
 0
 
The 2 excited state has energy E2 = +hω with eigenket | χ 2 >=| 1 − 1 >=  0  .
nd

1 
 
Solution: The eigenvalues of Sz are determined from
h−λ 0 0
0 −λ 0 = 0 and hence (h − λ )λ (h + λ ) = 0
0 0 −h−λ
Thus, there are three eigenvalues λ = h,0,−h . The eigenket of Sz corresponding to λ = 0 is
determined from
 1 0 0  a   a   0   0
        
h 0 0 0  b  = h 0  =  0  which implies that a = c = 0 and | 10 >=  1 
 0 0 − 1 c   − c   0   0
        
The eigenkets of Sz corresponding to λ = ±h is determined from
 1 0 0  a   a  a
      
h 0 0 0  b  = h 0  = ±h b  which implies that a = ±a and -c = ±c and hence
 0 0 − 1 c   − c  c
      
1   0
   
| 11 >=  0  and | 1 − 1 >=  0  and where S z | 1m >= mh | 1m > .
 0 1 
   
r r
The Hamiltonian is given by H = −γS ⋅ B = −γB0 S z = −ωS z . Hence there are three energy levels
E0 = −hω , E1 = 0 , E2 = +hω . The ground state (i.e. lowest) energy is E0 = −hω with eigenket

Department of Physics Page 6 of 25


PHY4604 Fall 2006 Final Exam Solutions

1 
 
| χ 0 >=| 11 >=  0  .
 0
 
The 1st excited state has energy E1 = 0 with eigenket
 0
 
| χ1 >=| 10 >= 1  .
 0
 
The 2nd excited state has energy E2 = +hω with eigenket
 0
 
| χ 2 >=| 1 − 1 >=  0  .
1 
 
(C) Suppose that at t = 0 the W+ boson is in the state
 1
1  
| χ (0) >=  1 .
3 
 1
(1) (2 points) What is | χ (t ) > ? Express your answer in terms of ω = γB0 .
 e + iωt 
1  
Answer: | χ (t ) >=  1 
3  − iωt 
e 
Solution: We know that
| χ (t ) >= c0 | χ 0 > e −iE0 t / h + c1 | χ1 > e − iE1t / h + c2 | χ 2 > e −iE 2 t / h
where
 1
1   1
c0 =< χ 0 | χ (0) >= (1 0 0)  1 =
3  3
 1
 1
1   1
c1 =< χ1 | χ (0) >= (0 1 0)  1 =
3  3
 1
 1
1   1
c2 =< χ 2 | χ (0) >= (0 0 1)  1 =
3  3
 1
Hence,

Department of Physics Page 7 of 25


PHY4604 Fall 2006 Final Exam Solutions

| χ (t ) >= c0 | χ 0 > e + iωt + c1 | χ1 > +c2 | χ 2 > e − iωt


1  0  0
1 + iωt   1   1 − iωt  
= e  0 + 1 + e  0
3  0 3  3 1
   0  
 e + iωt 
1  
=  1 
3  − iωt 
e 
where ω = γB0 .
(2) (2 points) If I measure the energy of the state | χ (t ) > , what values might I get and what is
the probability of getting these values.
Answer: We will measure E0 = −hω with probability P0 = 13 and we will measure E1 = 0 with
probability P1 = 13 and we will measure E2 = +hω with probability P2 = 13 .
Solution: We will measure E0 with probability P0 =| c0 |2 = 1
3
and we will measure E1 with
probability P1 =| c1 |2 = 13 and we will measure E2 with probability P2 =| c2 |2 = 13 .
(3) (2 points) What is the average energy, <E>, for the state | χ (t ) > ? Express your answer in
terms of ω = γB0 .
Answer: <E> = 0.
Solution: We can calculate <E> in two ways. It is given by
< E >= P0 E0 + P1E1 + P2 E2 = 13 (−hγB0 ) + 13 (0) + 13 (+hγB0 ) = 0 .
< E >=< χ (t ) | H | χ (t ) >= −ω < χ (t ) | S z | χ (t ) >
+ iωt
 1 0 0  e 
− hω − iωt   
3
e ( )
1 e + iωt  0 0 0  1 
 0 0 − 1 e − iωt 
  
+ iωt
 e 
− hω − iωt   − hω
=
3
e ( + iωt
1 e  0 = ) 3
(1 − 1) = 0
 − e − iωt 
 
(4) (2 points) What is the uncertainty in the energy, ∆E, for the state | χ (t ) > ? Express your
answer in terms of ω = γB0 .
Answer: ∆E = hω .
2
3

Solution: We can calculate <E2> in two ways. It is given by


< E 2 >= P0 E02 + P1E12 + P2 E22 = 13 (−hγB0 ) 2 + 13 (0) 2 + 13 (+hγB0 ) 2 = 23 (hγB0 ) 2 = 23 (hω ) 2
And it is also given by

Department of Physics Page 8 of 25


PHY4604 Fall 2006 Final Exam Solutions

< E 2 >=< χ (t ) | H 2 | χ (t ) >= ω 2 < χ (t ) | S z2 | χ (t ) >


+ iωt
 1 0 0  e 
(hω ) − iωt   
( 1 e + iωt ) 0 0 0  1 
2
e
3  0 0 1  e − iωt 
  
 e + iωt 
(hω ) 2 − iωt   (hω ) 2 2(hω ) 2
=
3
(e 1 e + iωt ) 0  =
3
(1 + 1) =
3
 e − iωt 
 
where I used S z2 from (a). Hence,
∆E = < E 2 > − < E > 2 = 2
3
( hω ) 2 = 2
3

(D) (4 points) What is <Sx(t)>, <Sy(t)>, and <Sz(t)> in the state | χ (t ) > ? Express your answer in
terms of ω = γB0 .
4h 4h
Answer: < S x (t ) >= cos(ωt ) , < S y (t ) >= − sin(ωt ) , < S z (t ) >= 0
3 2 3 2
Solution: We see that
+ i ωt
 0 1 0  e 
 
< S x (t ) >=< χ (t ) | S x | χ (t ) >=
h
(e−iωt 1 e+iωt ) 1 0 1  1 
3 2  0 1 0  e −iω0t 
  
 1 
 + iω t 
=
h
(e − i ωt
1 e + i ωt
)
e + e  =
−iωt h
(
e −iωt + e +iωt + e −iωt + e +iωt )
3 2   3 2
 1 
4h
= cos(ωt )
3 2
+ i ωt
 0 − 1 0  e 
  
< S y (t ) >=< χ (t ) | S y | χ (t ) >= e (
ih −iω0t
1 e + iω t ) 1 0 − 1  1 
3 2  0 1 0  e −iω0t 
  
 −1 
 + iω t 
=
ih −iωt
e (
1 e + i ωt ) e − e  =
−iωt ih
(
− e −iωt + e +iωt − e −iωt + e +iωt )
3 2   3 2
 1 
4h
=− sin(ωt )
3 2

Department of Physics Page 9 of 25


PHY4604 Fall 2006 Final Exam Solutions

+ i ωt
 1 0 0  e 
  
< S z (t ) >=< χ (t ) | S z | χ (t ) >=
h −iω0t
3
e (
1 e + i ωt )
 0 0 0  1 
 0 0 − 1 e −iω0t 
  
 e + i ωt 
  h
h
(
= e −iωt 1 e +iωt  0  = (1 − 1) = 0
3
)
 − e −iωt  3
 
(E) (5 points) What is ∆Sx(t), ∆Sy(t), and ∆Sz(t) in the state | χ (t ) > ? Express your answer in
terms of ω = γB0 .
h h 2
Answer: ∆S x (t ) = 3 − 2 cos 2 (ωt ) , ∆S y (t ) = 3 − 2 sin 2 (ωt ) , ∆S z (t ) = h.
3 3 3
Solution: We see that
1 0 1  e + iωt 
  
( )
2
h
< S x2 (t ) >=< χ (t ) | S x2 | χ (t ) >= e −iωt 1 e +iωt  0 2 0  1 
6 1
 0 1  e −iω0t 
 e +iωt + e −iωt 
  h2
=
h 2 −iωt
6
e ( 1 e +iωt  ) 2 = (
2e −iωt cos(ωt ) + 2 + 2e +iωt cos(ωt ) )
 e +iωt + e −iωt  6
 

( ) ( )
2
h h2
= (e +iωt + e −iωt ) cos(ωt ) + 1 = 2 cos 2 (ωt ) + 1
3 3

( ∆S x (t )) 2 =< S x2 (t ) > − < S x (t ) > 2 =


h2
3
(
2 cos 2 (ωt ) + 1 − 83 cos 2 (ωt ) )
=
h2
3
(
1 − 23 cos 2 (ωt ) =
h2
9
) (
3 − 2 cos 2 (ωt ) )
and hence
h h h
∆S x (t ) = 3 − 2 cos 2 (ωt ) = 1 + 2 sin 2 (ωt ) = 2 − cos(2ωt ) .
3 3 3
Also,
+ i ωt
 1 0 − 1 e 
  
( )
2
h −iωt
< S y2 (t ) >=< χ (t ) | S y2 | χ (t ) >= e 1 e + i ωt  0 2 0  1 
6  − 1 0 1  e −iω0t 
  
 e +iωt − e −iωt 
  h2
( ) ( )
2
h
= e −iωt 1 e +iωt  2 = 2ie −iωt sin(ωt ) + 2 − 2ie +iωt sin(ωt )
6  − e +iωt + e −iωt  6
 

=
h2
3
(− i (e +iωt − e −iωt ) sin(ωt ) + 1 = )h2
3
(
2 sin 2 (ωt ) + 1 )

Department of Physics Page 10 of 25


PHY4604 Fall 2006 Final Exam Solutions

( ∆S y (t )) 2 =< S y2 (t ) > − < S y (t ) > 2 =


h2
3
(
2 sin 2 (ωt ) + 1 − 83 sin 2 (ωt ) )
=
h2
3
(
1 − 23 sin 2 (ωt ) = )
h2
9
(
3 − 2 sin 2 (ωt ) )
and hence
h h h
∆S y (t ) = 3 − 2 sin 2 (ωt ) = 1 + 2 cos 2 (ωt ) = 2 + cos( 2ωt ) .
3 3 3
Also,
+ i ωt
 1 0 0  e 
  
( )
2
h −iωt
< S z2 (t ) >=< χ (t ) | S z2 | χ (t ) >= e 1 e + iω t  0 0 0  1 
3  0 0 1  e −iω0t 
  
 e + i ωt 
  h2
( )
2
h −iωt 2h 2
= e 1 e + iω t  0  = (1 + 1) =
3  e −iωt  3 3
 
2h 2
( ∆S z (t )) 2 =< S z2 (t ) > − < S z (t ) > 2 =
3
and hence
2
∆S z (t ) = h.
3

Problem 3 (25 points): Suppose we have two vector operators


r r r r
( J1 )op and ( J 2 )op with [( J1 )op , ( J 2 )op ] = 0
and each of the vectors obey the same SU(2) “lie algebra”:
[( J1i )op , ( J1 j )op ] = iε ijk ( J1k )op and [( J 2i )op , ( J 2 j )op ] = iε ijk ( J 2 k )op .
The states |j1m1> are the eigenkets of ( J12 ) op and ( J1z )op and the states |j2m2> are the eigenkets
of ( J 22 ) op and ( J 2 z )op as follows:

( J12 )op | j1m1 >= j1 ( j1 + 1) | j1m1 > ( J 22 )op | j2 m2 >= j2 ( j2 + 1) | j2 m2 >


( J1z )op | j1m1 >= m1 | j1m1 > ( J 2 z )op | j2 m2 >= m2 | j2 m2 >
Also we know that
( J1± )op | j1m1 >= j1 ( j1 + 1) − m1 (m1 ± 1) | j1m1 ± 1 >
( J 2± )op | j2 m2 >= j2 ( j2 + 1) − m2 (m2 ± 1) | j2 m2 ± 1 >
± ±
where ( J1 )op = ( J1x )op ± i ( J1 y )op and ( J 2 )op = ( J 2 x )op ± i ( J 2 y )op . Now consider the vector
sum of the two operators,
r r r
( J )op = ( J1 )op + ( J 2 )op or ( J i )op = ( J i1 )op + ( J i 2 )op for i = 1,2, 3.

Department of Physics Page 11 of 25


PHY4604 Fall 2006 Final Exam Solutions

(A) (1 point): Show that


r r r r r r r r
( J 2 )op = ( J )op ⋅ ( J )op = ( J1 + J 2 )op ⋅ ( J1 + J 2 )op = ( J12 )op + ( J 22 )op + 2( J1 )op ⋅ ( J 2 )op
= ( J12 )op + ( J 22 )op + ( J1+ )op ( J 2− )op + ( J1− )op ( J 2+ )op + 2( J1z )op ( J 2 z )op
Solution: First we note that
J1x = 12 ( J1+ + J1− ) J 2 x = 12 ( J 2+ + J 2− )
and
J1 y = 21i ( J1+ − J1− ) J2y = 1
2i
( J 2+ − J 2− )
Hence,
r r
J 2 = J12 + J 22 + 2 J1 ⋅ J 2 = J12 + J 22 + 2 J1x J 2 x + 2 J1 y J 2 y + 2 J1z J 2 z
= J12 + J 22 + 12 ( J1+ + J1− )( J 2+ + J 2− ) − 12 ( J1+ − J1− )( J 2+ − J 2− ) + 2 J1z J 2 z
= J12 + J 22 + J1+ J 2− + J1− J 2+ + 2 J1z J 2 z
(B) Now consider the case where j1 = 1 and j2 = 1 (i.e. 3 × 3) and define the states as follows:
| Y11 >1 =| 11 >1 | Y11 > 2 =| 11 > 2
| Y10 >1 =| 10 >1 and | Y10 > 2 =| 10 > 2
| Y1−1 >1 =| 1 − 1 >1 | Y1−1 > 2 =| 1 − 1 > 2
Consider the three superposition states
1 1 1
| ψ a >≡ | Y11 >1| Y1−1 > 2 − | Y10 >1| Y10 > 2 + | Y1−1 >1| Y11 > 2
3 3 3
1 1
| ψ b >≡ | Y11 >1| Y1−1 > 2 − | Y1−1 >1| Y11 > 2
2 2
1 2 1
| ψ c >≡ | Y11 >1| Y1−1 > 2 + | Y10 >1| Y10 > 2 + | Y1−1 >1| Y11 > 2
6 3 6
Calculate the following:
(1) (1 point) < ψ a | ψ a >=
(2) (1 point) < ψ b | ψ b >=
(3) (1 point) < ψ c | ψ c >=
(4) (1 point) < ψ a | ψ b >=
(5) (1 point) < ψ a | ψ c >=
(6) (1 point) < ψ b | ψ c >=

Calculate the following and express your answer in terms of |ψa>, |ψb>, and |ψc> :
(1) (1 point) ( J1 + J 2 ) | ψ a >=
2 2

(2) (1 point) ( J1 + J 2 ) | ψ b >=


2 2

(3) (1 point) ( J1 + J 2 ) | ψ c >=


2 2

(4) (1 point) J z | ψ a >=

Department of Physics Page 12 of 25


PHY4604 Fall 2006 Final Exam Solutions

(5) (1 point) J z | ψ b >=


(6) (1 point) J z | ψ c >=
(7) (1 point) ( J 1 J 2 + J1 J 2 + 2 J1z J 2 z ) | ψ a >=
+ − − +

(8) (1 point) ( J1 J 2 + J1 J 2 + 2 J1z J 2 z ) | ψ b >=


+ − − +

(9) (1 point) ( J1 J 2 + J1 J 2 + 2 J1z J 2 z ) | ψ c >=


+ − − +

(10) (2 points) J | ψ a >=


2

(11) (2 points) J | ψ b >=


2

(12) (2 points) J | ψ c >=


2

(3 points) Are the states |ψa>, |ψb>, and |ψc> eigenstates of the J2 and Jz and if so what are their
eigenvalues?
Answer:
(1) ( J1 + J 2 ) | ψ a >= 4 | ψ a >
2 2

(2) ( J1 + J 2 ) | ψ b >= 4 | ψ b >


2 2

(3) ( J1 + J 2 ) | ψ c >= 4 | ψ c >


2 2

(4) J z |ψ a >= 0 | ψ a >


(5) J z | ψ b >= 0 | ψ b >
(6) J z | ψ c >= 0 | ψ c >
(7) ( J1 J 2 + J1 J 2 + 2 J1z J 2 z ) | ψ a >= −4 | ψ a >
+ − − +

(8) ( J1 J 2 + J1 J 2 + 2 J1z J 2 z ) | ψ b >= −2 | ψ b >


+ − − +

(9) ( J1 J 2 + J1 J 2 + 2 J1z J 2 z ) | ψ c >= +2 | ψ c >


+ − − +

(10) J | ψ a >= 0 | ψ a >


2

(11) J | ψ b >= 2 | ψ b >


2

(12) J | ψ c >= 6 | ψ c >


2

The state |ψa> is an eigenstate of J2 and Jz with j = 0 and mj = 0 (i.e. |ψa>=|00>). The state |ψb>
is an eigenstate of J2 and Jz with j = 1 and mj = 0 (i.e. |ψb>=|10>). The state |ψc> is an eigenstate
of J2 and Jz with j = 2 and mj = 0 (i.e. |ψc>=|20>).
Solution: We know that
J12 | Y11 >1 = J12 | 11 >1 = 1(1 + 1) | 11 >1 = 2 | Y11 >1 J12 | Y11 >1 = J12 | 11 >1 = 1(1 + 1) | 11 >1 = 2 | Y11 >1
J12 | Y10 >1 = J12 | 10 >1 = 1(1 + 1) | 10 >1 = 2 | Y10 >1 J12 | Y10 >1 = J12 | 10 >1 = 1(1 + 1) | 10 >1 = 2 | Y10 >1
J12 | Y1−1 >1 = J12 | 1 − 1 >1 = 1(1 + 1) | 1 − 1 >1 = 2 | Y1−1 >1 J12 | Y1−1 >1 = J12 | 1 − 1 >1 = 1(1 + 1) | 1 − 1 >1 = 2 | Y1−1 >1
J1z | Y11 >1 = J1z | 11 >1 = 1 | 11 >1 = 1 | Y11 >1 J1z | Y11 >1 = J1z | 11 >1 = 1 | 11 >1 = 1 | Y11 >1
J1z | Y11 >1 = J1z | 10 >1 = 0 | 10 >1 = 0 J1z | Y11 >1 = J1z | 10 >1 = 0 | 10 >1 = 0
J1z | Y1−1 >1 = J1z | 1 − 1 >1 = −1 | 1 − 1 >1 = −1 | Y1−1 >1 J1z | Y1−1 >1 = J1z | 1 − 1 >1 = −1 | 1 − 1 >1 = −1 | Y1−1 >1

Department of Physics Page 13 of 25


PHY4604 Fall 2006 Final Exam Solutions

J1+ | Y11 >1 = J1+ | 11 >1 = 0


J1+ | Y10 >1 = J1+ | 10 >1 = 1(1 + 1) − 0(0 + 1) | 11 >1 = 2 | Y11 >1
J1+ | Y1−1 >1 = J1+ | 1 − 1 >1 = 1(1 + 1) + 1(−1 + 1) | 10 >1 = 2 | Y10 >1
J1− | Y11 >1 = J1− | 11 >1 = 1(1 + 1) − 1(1 − 1) | 10 >1 = 2 | Y10 >1
J1− | Y10 >1 = J1− | 10 >1 = 1(1 + 1) − 0(0 − 1) | 1 − 1 >1 = 2 | Y1−1 >1
J1− | Y1−1 >1 = J1− | 1 − 1 >1 = 0
and hence
 1 1 1 
( J12 + J 22 ) | ψ a >= ( J12 + J 22 ) | 11 >1| 1 − 1 > 2 − | 00 >1| 00 > 2 + | 1 − 1 >1| 11 > 2 
 3 3 3 
= (2 + 2) | ψ a >= 4 | ψ a >
 1 1 
( J12 + J 22 ) | ψ b >= ( J12 + J 22 ) | 11 >1| 1 − 1 > 2 − | 1 − 1 >1| 11 >2 
 2 2 
= (2 + 2) | ψ b >= 4 | ψ b >
 1 2 1 
( J12 + J 22 ) | ψ c >= ( J12 + J 22 ) | 11 >1| 1 − 1 > 2 + | 00 >1| 00 > 2 + | 1 − 1 >1| 11 > 2 
 6 3 6 
= (2 + 2) | ψ c >= 4 | ψ c >
 1 1 1 
J z | ψ a >= ( J1z + J 2 z ) | 11 >1| 1 − 1 > 2 − | 00 >1| 00 > 2 + | 1 − 1 >1| 11 > 2 
 3 3 3 
1 1 1
= (1 − 1) | 11 >1| 1 − 1 >2 −(0 + 0) | 00 >1| 00 > 2 +(−1 + 1) | 1 − 1 >1| 11 > 2
3 3 3
= 0 |ψ a >
 1 1 
J z | ψ b >= ( J1z + J 2 z ) | 11 >1| 1 − 1 > 2 − | 1 − 1 >1| 11 > 2 
 2 2 
1 1
= (1 − 1) | 11 >1| 1 − 1 > 2 −(−1 + 1) | 1 − 1 >1| 11 > 2
2 2
= 0 |ψ b >
 1 2 1 
J z | ψ c >= ( J1z + J 2 z ) | 11 >1| 1 − 1 >2 + | 00 >1| 00 > 2 + | 1 − 1 >1| 11 > 2 
 6 3 6 
1 2 1
= (1 − 1) | 11 >1| 1 − 1 > 2 +(0 + 0) | 00 >1| 00 > 2 +(−1 + 1) | 1 − 1 >1| 11 > 2
6 3 3
= 0 |ψ c >

Department of Physics Page 14 of 25


PHY4604 Fall 2006 Final Exam Solutions

( J1+ J 2− + J1− J 2+ + 2 J1z J 2 z ) | ψ a >


 1 1 1 
= ( J1+ J 2− + J1− J 2+ + 2 J1z J 2 z ) | 11 >1| 1 − 1 > 2 − | 10 >1| 10 >2 + | 1 − 1 >1| 11 > 2 
 3 3 3 
 1 1 1 
= ( J1+ J 2− ) | 11 >1| 1 − 1 >2 − | 10 >1| 10 > 2 + | 1 − 1 >1| 11 > 2 
 3 3 3 
 1 1 1 
+ ( J1− J 2+ ) | 11 >1| 1 − 1 > 2 − | 10 >1| 10 >2 + | 1 − 1 >1| 11 > 2 
 3 3 3 
 1 1 1 
+ (2 J1z J 2 z ) | 11 >1| 1 − 1 > 2 − | 10 >1| 10 >2 + | 1 − 1 >1| 11 > 2 
 3 3 3 
1 1
=− 2 2 | 11 >1| 1 − 1 >2 + 2 2 | 10 >1| 10 >2
3 3
1 1
+ 2 2 | 10 >1| 10 >2 − 2 2 | 1 − 1 >1| 11 > 2
3 3
1 1 1
+ (2)(1)(−1) | 11 >1| 1 − 1 >2 −(2)(0)(0) | 10 >1| 10 >2 +(2)(−1)(1) | 1 − 1 >1| 11 >2
3 3 3
1 1 1
= (−2 − 2) | 11 >1| 1 − 1 > 2 +(2 + 2) | 10 >1| 10 > 2 +(−2 − 2) | 1 − 1 >1| 11 > 2
3 3 3
= −4 | ψ a >
and hence
J 2 | ψ a >= ( J12 + J 22 + J1+ J 2− + J1− J 2+ + 2 J1z J 2 z ) | ψ a >= (4 − 4) | ψ a >= 0 | ψ a > .
Thus, the state |ψa> is an eigenstate of J2 and Jz with j = 0 and mj = 0 (i.e. |ψa>=|00>).
( J1+ J 2− + J1− J 2+ + 2 J1z J 2 z ) | ψ b >
 1 1 
= ( J1+ J 2− + J1− J 2+ + 2 J1z J 2 z ) | 11 >1| 1 − 1 > 2 − | 1 − 1 >1| 11 > 2 
 2 2 
 1 1 
= ( J1+ J 2− ) | 11 >1| 1 − 1 > 2 − | 1 − 1 >1| 11 > 2 
 2 2 
 1 1 
+ ( J1− J 2+ ) | 11 >1| 1 − 1 > 2 − | 1 − 1 >1| 11 > 2 
 2 2 
 1 1 
+ (2 J1z J 2 z ) | 11 >1| 1 − 1 > 2 − | 1 − 1 >1| 11 > 2 
 2 2 
1
=− 2 2 | 10 >1| 10 > 2
2
1
+ 2 2 | 10 >1| 10 > 2
2
1 1
+ (2)(1)(−1) | 11 >1| 1 − 1 > 2 −(2)(−1)(1) | 1 − 1 >1| 11 > 2
2 2
1 1 1
= (−2) | 11 >1| 1 − 1 > 2 +(−2 + 2) | 10 >1| 10 > 2 −(−2) | 1 − 1 >1| 11 > 2
2 2 3
= −2 | ψ b >
and hence
J 2 | ψ b >= ( J12 + J 22 + J1+ J 2− + J1− J 2+ + 2 J1z J 2 z ) | ψ b >= (4 − 2) | ψ b >= 2 | ψ b > .

Department of Physics Page 15 of 25


PHY4604 Fall 2006 Final Exam Solutions

Thus, the state |ψb> is an eigenstate of J2 and Jz with j = 1 and mj = 0 (i.e. |ψb>=|10>).
( J1+ J 2− + J1− J 2+ + 2 J1z J 2 z ) | ψ c >
 1 2 1 
= ( J1+ J 2− + J1− J 2+ + 2 J1z J 2 z ) | 11 >1| 1 − 1 > 2 + | 10 >1| 10 > 2 + | 1 − 1 >1| 11 > 2 
 6 3 6 
 1 2 1 
= ( J1+ J 2− ) | 11 >1| 1 − 1 > 2 + | 10 >1| 10 > 2 + | 1 − 1 >1| 11 > 2 
 6 3 6 
 1 2 1 
+ ( J1− J 2+ ) | 11 >1| 1 − 1 > 2 + | 10 >1| 10 > 2 + | 1 − 1 >1| 11 > 2 
 6 3 6 
 1 2 1 
+ (2 J1z J 2 z ) | 11 >1| 1 − 1 > 2 + | 10 >1| 10 > 2 + | 1 − 1 >1| 11 > 2 
 6 3 6 
2 1
= 2 2 | 11 >1| 1 − 1 > 2 + 2 2 | 10 >1| 10 > 2
3 6
1 2
+ 2 2 | 10 >1| 10 > 2 + 2 2 | 1 − 1 >1| 11 > 2
6 3
1 2 1
+ (2)(1)(−1) | 11 >1| 1 − 1 > 2 +(2)(0)(0) | 10 >1| 10 > 2 +(2)(−1)(1) | 1 − 1 >1| 11 > 2
6 3 6
1 2 1
= (4 − 2) | 11 >1| 1 − 1 > 2 +(1 + 1) | 10 >1| 10 > 2 +(4 − 2) | 1 − 1 >1| 11 > 2
6 3 6
= 2 |ψ c >
and hence
J 2 | ψ c >= ( J12 + J 22 + J1+ J 2− + J1− J 2+ + 2 J1z J 2 z ) | ψ c >= (4 + 2) | ψ c >= 6 | ψ c > .
Thus, the state |ψc> is an eigenstate of J2 and Jz with j = 2 and mj = 0 (i.e. |ψc>=|20>).

Two Particles in a Box


Problem 4 (25 points): Consider the case of two non-interacting particles
both with mass m in a one-dimensional infinite square well given by
V(x) = 0 for 0 < x < L, and V(x) = ∞. For one particle we know that the
stationary states of Schrödinger’s equation are given by ψ(x2)
Ψn ( x, t ) = ψ n ( x)e −iE n t / h and En = n 2 E0 , and n is a positive integer and
ψ(x1)
π h
2 2
2
E0 = 2
and where ψ n ( x) = sin( nπx / L) . For two (non-interacting)
2mL L
particles we look for a solution of the form
0 L
Ψ ( x1 , x 2 , t ) = ψ ( x1 , x 2 )e − iEt / h = ψ ( x1 )ψ ( x 2 )e − iEt / h with
( p x )12 ( p x ) 22
+ =E.
2m 2m
(A) (1 point): Show that ψ αβ ( x1 , x2 ) = ψ α ( x1 )ψ β ( x2 ) , is a solution to the two particle non-
interaction Schrödinger equations, where α and β are positive integers and ψα(x) and ψβ(x) are
the one particle stationary state solutions. Show that the allowed energy levels are given by
Eαβ = (α 2 + β 2 ) E0 .
Solution: We see that

Department of Physics Page 16 of 25


PHY4604 Fall 2006 Final Exam Solutions

( p x )12 ( p x ) 22
Ψ ( x1 , x 2 , t ) = ψ ( x1 , x 2 )e − iEt / h
= ψ ( x1 )ψ ( x 2 )e − iEt / h
with + =E.
2m 2m
Thus,
h 2 d 2ψ ( x1 , x2 ) h 2 d 2ψ ( x1 , x 2 )
− − = Eψ ( x1 , x 2 )
2m dx12 2m dx 22
h2 d 2ψ ( x1 ) h 2 d 2ψ ( x 2 )
− ψ ( x2 ) − ψ ( x1 ) = Eψ ( x1 )ψ ( x2 )
2m dx12 2m dx 22
h 2 1 d 2ψ ( x1 ) h 2 1 d 2ψ ( x 2 )
− − =E
2m ψ ( x1 ) dx12 2m ψ ( x 2 ) dx 22
Hence, E = E1 + E2 and
h 2 d 2ψ ( x1 ) h 2 d 2ψ ( x 2 )
− = E 1ψ ( x1 ) − = E 2ψ ( x 2 )
2m dx12 2m dx 22
The total energy is therefore given by
h 2π 2 (α 2 + β 2 ) h 2π 2α 2 h 2π 2 β 2
Eαβ = ( E1 ) α + ( E 2 ) β = ( E )
1 α = ( E )
2 β =
2 mL2 2mL2 2 mL2
where α = 1, 2, 3, ... and β = 1, 2, 3, ... and
2
ψ αβ ( x1 , x 2 ) = ψ α ( x1 )ψ β ( x 2 ) = sin(απx1 / L ) sin( βπ x 2 / L )
L
4
ρ αβ ( x1 , x 2 ) = ψ α* ( x1 )ψ β* ( x 2 )ψ α ( x1 )ψ β ( x 2 ) = 2
sin 2 (απ x1 / L ) sin 2 ( βπ x 2 / L )
L
This solution corresponds to the case where the two particles are distinguishable. Define the
following three probabilities. Let PαβLL be the probability of finding both particles in the left 1/3
of the box as follows:
L/3L/3
PαβLL = ∫ ∫ | ψ αβ ( x , x ) |
2
1 2 dx1dx2 .
0 0
LR
Let Pαβ be the probability of finding one particle in the left 1/3 of the box and one particle in the
right 1/3 of the box as follows:
L/3 L
PαβLR = ∫ ∫ | ψ αβ ( x , x ) |
2
1 2 dx1dx2 .
0 2L / 3
CC
Let Pαβ be the probability of finding both particles in the center 1/3 of the box as follows:
2 L / 32 L / 3
Pαβ = ∫ ∫ |ψ αβ ( x , x ) |
CC 2
1 2 dx1dx2 .
L/3 L/3

Calculate the following observables for the ground state and the first excited state for the
distinguishable case. (Show your work and fill in the table)
Points Observable Ground State 1st Excited State
2 Eαβ
2 PαβLL

Department of Physics Page 17 of 25


PHY4604 Fall 2006 Final Exam Solutions

2 PαβLR
2 PαβCC
Answer: Distinguishable
Observable Ground State α = 1 β = 1 1st Excited State α = 1 β = 2
Eαβ 2E0 5E0
2

PαβLL (P ) L 2
1
=  −
3
 ≈ 0.0382

(P )(P ) =  13 − 4π3  13 + 8π3  ≈ 0.0786
1
L
2
L

 3 4π 
1
  
2

PαβLR (P )(P ) =  13 − 4π3 


1
L
1
R
≈ 0.0382 (P )(P ) =  13 − 4π3  13 + 8π3  ≈ 0.0786
1
L
2
R

    
2

PαβCC (P ) C 2
1
=  +
3
 ≈ 0.3709

(P )(P ) =  13 + 2π3  13 − 4π3  ≈ 0.1191
1
C
2
C

 3 2π 
1
  
Solution: Let me first work on some integrals that I will need
π /3 π /3
 2  π sin( 2π / 3) 
L/3
2 2L 2  L  y sin( 2 y ) 
P = ∫0 sin (πx / L)dx = L  π  ∫0 sin ( y)dx = L  π  2 − 4  0 =   −
L 2 2

 π  6
1
L 4 
1 sin( 2π / 3) 1 3
= − = −
3 2π 3 4π
2π / 3 2π / 3
 2  2π sin( 4π / 3) 
2L / 3
2 2L 2  L  y sin( 2 y ) 
P = ∫0 sin (πx / L)dx = L  π  ∫0 sin ( y)dx = L  π  2 − 4  0 =   −
T 2 2

 π  6
1
L 4 
2 sin( 4π / 3) 2 3
= − = +
3 2π 3 4π

1 3
P1C = P1T − P1L = +
3 2π
L L/3
2 2
P1R = ∫ sin 2 (πx / L)dx = ∫ sin (πx / L)dx = P1L
2

L 2L / 3 L 0

Note that P + P + P = 1 .
1
L
1
C
1
R

2π / 3 2π / 3
 1  π sin( 4π / 3) 
L/3
2 2 L  2  L  y sin( 2 y ) 
P = ∫0 sin (2πx / L)dx = L  2π  ∫0 sin ( y)dx = L  2π  2 − 4  0 =   −
L 2 2

 π  3
2
L 4 
1 sin( 4π / 3) 1 3
= − = +
3 4π 3 8π
4π / 3 4π / 3
 1  4π sin(8π / 3) 
2L / 3
2 2 L  2  L  y sin( 2 y ) 
P = ∫0 sin (2πx / L)dx = L  2π  ∫0 sin ( y)dx = L  2π  2 − 4  0 =   −
T 2 2

 π  6
2
L 4 
2 sin(8π / 3) 2 3
= − = −
3 4π 3 8π

Department of Physics Page 18 of 25


PHY4604 Fall 2006 Final Exam Solutions

1 3
P2C = P2T − P2L = −
3 4π
L L/3
2 2
P = ∫ sin 2 (2πx / L)dx = ∫ sin (2πx / L)dx = P2L
R 2
2
L 2L / 3 L 0

Note that P + P + P = 1 .2
L
2
C
2
R

π π
 2  π  1
L/3
2 2 L  2  L  y sin(2 y ) 
P = ∫0 sin (3πx / L)dx = L  3π ∫0 sin ( y)dx = L  3π  2 − 4  0 =  3π  2  = 3
L 2 2
3
L
2L / 3 2π 2π
2 2 L  2  L  y sin( 2 y )   2  2
P = ∫0 sin (3πx / L)dx = L  3π  ∫0 sin ( y)dx = L  3π  2 − 4  0 =  3π (π ) = 3
T 2 2
3
L
1
P3C = P3T − P3L =
3
L L/3
2 2
L 2 L∫/ 3
P3 =
R
sin (3πx / L)dx = ∫ sin 2 (3πx / L)dx = P3L
2

L 0
Note that P3L + P3C + P3R = 1 . These results are summarized in the following table.
n=1 n=2 n=3
1 3 1 3 1
PnL − ≈ 0.1955 + ≈ 0.4022
3 4π 3 8π 3
1 3 1 3 1
PnC + ≈ 0.6090 − ≈ 0.1955
3 2π 3 4π 3
1 3 1 3 1
PnR − ≈ 0.1955 + ≈ 0.4022
3 4π 3 8π 3
sum 1 1 1

We will also need the following integrals.


L/3 L/3
2 1
I12 = ∫ sin(πx / L) sin(2πx / L)dx = ∫ (cos(3πx / L) − cos(πx / L) )dx =
L

L 0 L 0
L/3 L/3
1 1 1 1 3
∫ cos(πx / L)dx = 3π (sin( y)) (sin( y) ) π0 / 3 = −
π

L ∫ cos(3πx / L)dx −
0
L 0
0

π 2π
2L / 3 2L / 3
2 1
I12C =
L ∫ sin(πx / L) sin(2πx / L)dx = L ∫ (cos(3πx / L) − cos(πx / L))dx =
L/3 L/3
2L / 3 2L / 3
1 1 1 1
∫ cos(3πx / L)dx − L ∫ cos(πx / L)dx = 3π (sin( y))π (sin( y ) ) π2π/ 3/ 3 = 0


L L/3 L/3
π
L L
2 1
∫ (cos(3πx / L) − cos(πx / L) )dx =
L 2 L∫/ 3
I12R = sin(πx / L) sin(2πx / L)dx =
L 2L / 3
L L
1 1 1
∫ cos(3πx / L)dx − ∫ cos(πx / L)dx = (sin( y ) ) 32ππ − 1 (sin( y ) ) π2π / 3 = + 3
L 2L / 3 L 2L / 3 3π π 2π

Department of Physics Page 19 of 25


PHY4604 Fall 2006 Final Exam Solutions

L/3 L/3
2 1
I = ∫0 sin(πx / L) sin(3πx / L)dx = L ∫ (cos(4πx / L) − cos(2πx / L))dx =
L
13
L 0
L/3 L/3
1 1 1 1
∫ cos(4πx / L)dx − L ∫ cos(2πx / L)dx = 4π (sin( y)) (sin( y ) ) 02π / 3
4π / 3

L 0 0
0

3 3 3 3
=− − =−
8π 4π 8π
2L / 3 2L / 3
2 1
(cos(4πx / L) − cos(2πx / L) )dx =
L L∫/ 3 L L∫/ 3
I13C = sin(πx / L ) sin( 3πx / L ) dx =

2L / 3 2L / 3
1 1 1 1
∫ cos(2πx / L)dx = 4π (sin( y)) π (sin( y ) ) 42ππ // 33
8π / 3

L ∫L / 3cos(4πx / L)dx − L L/3


4 /3


1  3 3 1  3 3 3 3 3 3
=  + − − − = + =
4π  2 2  2π  2 
2  4π 2π 4π
  
L L
2 1
∫ (cos(4πx / L) − cos(2πx / L))dx =
L 2 L∫/ 3
I13R = sin(πx / L) sin(3πx / L)dx =
L 2L / 3
L L
1 1 1
∫ cos(4πx / L)dx − ∫ cos(2πx / L)dx = (sin( y )) 84ππ / 3 − 1 (sin( y )) 24ππ / 3
L 2L / 3 L 2L / 3 4π 2π
3 3
3 3
=− − =−
8π 4π 8π
The results are summerized in the following table.
12 13
3 3 3
IL − ≈ −0.2757 − ≈ −0.2067
2π 8π
3 3
IC 0 + ≈ 0.4135

3 3 3
IR + ≈ +0.2757 − ≈ −0.2067
2π 8π
sum 0 0

Now for distinguishable particles we have


2
ψ αβ ( x1 , x2 ) = ψ α ( x1 )ψ β ( x2 ) = sin(απ x1 / L ) sin( βπ x2 / L )
L
4
ραβ
D
( x1 , x 2 ) =| ψ αβ ( x1 , x 2 ) | 2 =
sin 2 (απx1 / L) sin 2 ( βπ x 2 / L )
L2
The ground state is the state with α = 1 and β = 1, Thus, E0D = 2 E0 and

Department of Physics Page 20 of 25


PHY4604 Fall 2006 Final Exam Solutions

L/3L/3
4
(P ) = 2 ∫ ∫ sin (πx1 / L) sin 2 (πx2 / L)dx1dx2
LL 2
11 D
L 0 0
2
L/3 L/3
1 3
2
L
2 2
∫0 sin (πx1 / L)dx1 L ∫ sin
2
( )
(πx2 / L)dx2 = P
1
L 2
=  −
3 4π


0  
and
2 L / 32 L / 3
4
( P11CC ) D = ∫ ∫ sin (πx1 / L) sin 2 (πx2 / L)dx1dx2
2

L2 L/3 L/3
2
2L / 3 2L / 3
1 3
2
L
2 2
∫L / 3sin (πx1 / L)dx1 L ∫ sin
2
( )
(πx2 / L)dx2 = P
1
C 2
=  +
3 2π


L/3  
and
2L / 3 L
4
(P ) = 2 ∫ ∫ sin (πx1 / L) sin 2 (πx2 / L)dx1dx2
LR 2
11 D
L 0 2L / 3
2
2L / 3 L
1 3
2
L
2 2
∫0 sin (πx1 / L)dx1 L 2 L∫/sin
2
(πx2 / L)dx2 = P1L P1R = P1L ( ) 2
=  −
3 4π


3  

The 1st excited state has α = 1 and β = 2, Thus, E1D = 5E0 and
L/3L/3
4
( P12LL ) D = ∫ ∫ sin (πx1 / L) sin 2 (2πx2 / L)dx1dx2
2

L2 0 0
L/3 L/3
1 3  1 3
(2πx2 / L)dx2 = (P1L )(P2L ) =  −
2 2
∫0 sin (πx1 / L)dx1 L ∫ sin  + 
2 2
 
L 0  3 4π  3 8π 
and
2 L / 32 L / 3
4
( P12CC ) D = ∫ ∫ sin (πx1 / L) sin 2 (2πx2 / L)dx1dx2
2

L2 L/3 L/3
2L / 3 2L / 3
1 3  1 3
(2πx2 / L)dx2 = (P1C )(P2C ) =  +
2 2
∫L / 3sin (πx1 / L)dx1 L ∫ sin  − 
2 2

L 3 2π  3 4π 
L/3   
and
2L / 3 L
4
( P12LR ) D = ∫ ∫ sin (πx1 / L) sin 2 (2πx2 / L)dx1dx2
2

L2 0 2L / 3

2
2L / 3
2
L
1 3  1 3
∫0 sin (πx1 / L)dx1 L 2 L∫/sin (2πx2 / L)dx2 = P1L P2R =  −  + 
2 2

L 3 4π  3 8π 
3   

(B) For two identical bosons (i.e. particles with integral spins in the same spin state) we must use
the symmetric wavefunction

ψ αβ
S
( x1 , x 2 ) =
1
(ψ αβ ( x1 , x2 ) + ψ αβ ( x2 , x1 ) ) (α ≠ β symmetric under 1↔2)
2

Department of Physics Page 21 of 25


PHY4604 Fall 2006 Final Exam Solutions

1
ψ αα
S
(ψ αα ( x1 , x2 ) + ψ αα ( x2 , x1 ) ) (α = β symmetric under 1↔2)
( x1 , x 2 ) =
2
Calculate the following observables for the ground state and the first excited state for the
identical boson case. (Show your work and fill in the table)
Points Observable Ground State 1st Excited State
2 Eαβ
2 PαβLL
2 PαβLR
2 PαβCC
Answer: Bosons
Observable Ground State α = 1 β = 1 1st Excited State α = 1 β = 2
Eαβ 2E0 5E0
2
1 3  1 3  3
PαβLL (P ) L 2
1
=  −
3
 ≈ 0.0382
2
( )( ) ( )
P1 P + I
L L L 2
=  −  +

+
 


 3 4π  3 8π   2π 
2 12

 3 4π 
1

≈ 0.0786 + 0.0760 ≈ 0.1546


2
1 3  1 3  3
( )( ) ( )( )
2

(P )(P ) =  13 − 4π3  P P + I I =  −  + − 


L R L R
PαβLR L R
≈ 0.0382    
 3 4π  3 8π   2π 
1 2 12 12
1 1
 
≈ 0.0786 - 0.0760 ≈ 0.0026
1 3  1 3
CC
Pαβ (P ) C 2
1
=  +
3
2

 ≈ 0.3709
( )( ) ( ) 2
P1C P2C + I12C =  +
3 2 π
 −
 3 4π


1
3 2π    
 
≈ 0.1191
Solution: In this case we have
2
ψ αβ
S
( x1 , x2 ) = [sin(απx1 / L) sin( βπ x2 / L ) + sin( βπ x1 / L) sin(απx2 / L )] (α ≠ β)
L
2
ψ αα
S
( x1 , x2 ) = sin(απx1 / L ) sin(απx2 / L )
L
and

ραβ
BE
( x1 , x2 ) = ραβ
classical
( x1 , x2 ) + ραβ
int
( x1 , x2 ) (α ≠ β)
where
ραβ
int
( x1 , x2 ) ≡ Re (ψ αβ ( x1 , x2 )ψ βα

( x1 , x2 ) ) (α ≠ β)
and
ραα
BE
( x1 , x2 ) = ραα
classical
( x1 , x2 ) .
where
ραβ
classical
( x1 , x2 ) =
L
2
2
(
sin 2 (απx1 / L) sin 2 ( βπx2 / L) + sin 2 ( βπx1 / L) sin 2 (απx2 / L) )
4
ραβ
int
( x1 , x2 ) = 2 (sin(απx1 / L) sin(βπx1 / L) sin(απx2 / L) sin( βπx2 / L) )
L

Department of Physics Page 22 of 25


PHY4604 Fall 2006 Final Exam Solutions

and
ραβ
classical
( x1 , x2 ) = 12 ( ραβ
D
( x1 , x2 ) + ραβ
D
( x2 , x1 )) .
The ground state is the state with α = 1 and β = 1, Thus, E0BE = 2 E0 , and
( P11LL ) BE = ( P11LL ) D , ( P11CC ) BE = ( P11CC ) D , ( P11LR ) BE = ( P11LR ) D .

The 1st excited state has α = 1 and β = 2, Thus, E1D = 5E0 and

L/3L/3 L/3L/3 L/3L/3


( P12LL ) BE = ∫ ∫ ρ12 ( x1 , x2 )dx1dx2 = ∫ ∫ ρ12 ( x1 , x2 )dx1dx2 + ∫ ∫ρ
BE classical int
12 ( x1 , x2 )dx1dx2
0 0 0 0 0 0
2
1 3  1 3  3
( )( ) ( )
= P1L P2L + I12L
2
=  −
3 4π
 +   
 3 8π  +  2π 
    
where I used

L/3L/3 L/3L/3
4
∫ ∫ ρ12 ( x1, x2 )dx1dx2 = ∫ ∫ sin(πx / L) sin(2πx / L) sin(πx / L) sin(2πx2 / L)dx1dx2
int
1 1 2
0 0
L2 0 0
L/3 L/3
/ L) sin(2πx2 / L)dx2 = (I12L )
2 2 2
=
L ∫0 sin(πx1 / L) sin(2πx1 / L)dx1 L ∫ sin(πx
0
2

Also,
L/3 L L/3 L L/3 L
= ∫ ∫ρ ( x1 , x2 )dx1dx2 = ∫ ∫ρ ( x1 , x2 )dx1dx2 + ∫ ∫ρ
LR BE classical int
(P )12 BE 12 12 12 ( x1 , x2 )dx1dx2
0 2L / 3 0 2L / 3 0 2L / 3
2
1 3  1 3  3
= 1
(P )(P ) + (P )(P ) + (I )(I ) = (P )(P ) + (I )(I )
L R 1 L R L R L R L R
=  −  +

−
 


 3 4π  3 8π   2π 
2 1 2 2 2 1 12 12 1 2 12 12

where I used

L/3 L L/3 L
4
∫ ∫ ρ ( x1 , x2 )dx1dx2 = 2 ∫ ∫ sin(πx / L) sin(2πx / L) sin(πx / L) sin(2πx2 / L)dx1dx2
int
12 1 1 2
0 2L / 3
L 0 2L / 3
L/3 L
sin(πx2 / L) sin(2πx2 / L)dx2 = (I12L )(I12R )
2 2
=
L ∫ sin(πx1 / L) sin(2πx1 / L)dx1
0
L 2 L∫/ 3
Also,
2 L / 32 L / 3 2 L / 32 L / 3 2 L / 32 L / 3
( P12CC ) BE = ∫ ∫ ρ12 ( x1, x2 )dx1dx2 = ∫ ∫ ρ12 ( x1, x2 )dx1dx2 + ∫ ∫ρ
BE classical int
12 ( x1 , x2 )dx1dx2
L/3 L/3 L/3 L/3 L/3 L/3

1 3  1 3
= 1
2
(P )(P ) + (P )(P ) + (I ) = (P )(P ) + (I )
1
C
2
C 1
2 2
C
1
C C 2
12 1
C
2
C C 2
12 =  +
3 2π
 −
 3 4π


  
where I used

Department of Physics Page 23 of 25


PHY4604 Fall 2006 Final Exam Solutions

2 L / 32 L / 3 L / 3 2L / 3
4
∫ ∫ ρ12 ( x1 , x2 )dx1dx2 = ∫ ∫ sin(πx / L) sin(2πx / L) sin(πx / L) sin(2πx2 / L)dx1dx2
int
1 1 2
L/3 L/3
L2 L/3 L/3
2L / 3 2L / 3
/ L) sin(2πx2 / L)dx2 = (I12C )
2 2 2
=
L ∫L / 3sin(πx1 / L) sin(2πx1 / L)dx1 L ∫ sin(πx
L/3
2

(C) For two identical fermions (i.e. particles with half-integral spins in the same spin state) we
must use the symmetric wavefunction

(ψ αβ ( x1 , x2 ) −ψ αβ ( x2 , x1 ) ) (antisymmetric under 1↔2)


ψ αβA ( x1 , x 2 ) =
1
2
Calculate the following observables for the ground state and the first excited state for the
identical fermion case. (Show your work and fill in the table)
Points Observable Ground State 1st Excited State
2 Eαβ
2 PαβLL
2 PαβLR
2 PαβCC
Answer: Fermions
Observable Ground State α = 1 β = 2 1st Excited State α = 1 β = 3
Eαβ 5E0 10 E0
2 2
1 3  1 3  3 1 3  1   3 3 
PαβLL
( )( ) ( )
P P − I
L L
=  − 
L 2
 + −
 

 ( )( ) ( )
P P − I
L L L 2
=  −   − 
 


 3 4π  3 8π   2π   3 4π  3   8π 
1 2 12 1 3 13

≈ 0.0786 - 0.0760 ≈ 0.0026 ≈ 0.0652 - 0.0427 ≈ 0.0224


2
1 3  1   3 3 
(P )(P ) − (I )(I ) =  13 − 4π3  13 + 8π3  +  2π3 
L R L R
2
( )( ) ( )( )
P1 P − I I =  −
L R L R
  − 
 


 3 4π  3   8π 
3 13 13
PαβLR 1 2

12 12
   
≈ 0.0652 - 0.0427 ≈ 0.0224
≈ 0.0786 + 0.0760 ≈ 0.1546
2
1 3  1 3 1 3  1   3 3 
PαβCC
(P )(P ) + (I )
C C C 2
=  +  −


 ( )( ) ( )
P P − I
C C C 2
=  +   − 
 3   8π 

 3 2π  3 4π 
1 2 12 1 3 13
 3 2π   
≈ 0.1191 ≈ 0.2030 - 0.1710 ≈ 0.0320
Solution: In this case we have
2
ψ αβA ( x1 , x2 ) = [sin(απx1 / L ) sin( βπ x2 / L) − sin( βπ x1 / L) sin(απx2 / L )]
L
and

ραβ
FD
( x1 , x2 ) = ραβ
classical
( x1 , x2 ) − ραβ
int
( x1 , x2 )
where
ραβ
int
( x1 , x2 ) ≡ Re (ψ αβ ( x1 , x2 )ψ βα

( x1 , x2 ) ) (α ≠ β).

Department of Physics Page 24 of 25


PHY4604 Fall 2006 Final Exam Solutions

The ground has α = 1 and β = 2, Thus, E0FD = 5E0 and

L/3 L/3 L/3 L/3 L/3 L/3


( P12LL ) FD = ∫ ∫ ρ12 ( x1, x2 )dx1dx2 = ∫ ∫ ρ12 ( x1, x2 )dx1dx2 − ∫ ∫ρ
FD classical int
12 ( x1 , x2 )dx1dx2
0 0 0 0 0 0
2
1 3  1 3  3
= P ( )(P ) − (I )
1
L
2
L L 2
12 =  −
3 4π
 +   
 3 8π  −  2π 
    
Also,
L/3 L L/3 L L/3 L
= ∫ ∫ρ ( x1 , x2 )dx1dx2 = ∫ ∫ρ ( x1 , x2 )dx1dx2 − ∫ ∫ρ
LR FD classical int
(P )12 FD 12 12 12 ( x1 , x2 )dx1dx2
0 2L / 3 0 2L / 3 0 2L / 3
2
1 3  1 3  3
= 1
(P )(P ) + (P )(P ) − (I )(I ) = (P )(P ) − (I )(I )
L R 1 L R L R L R L R
=  −  +

+
 


 3 4π  3 8π   2π 
2 1 2 2 2 1 12 12 1 2 12 12

Also,
2 L / 32 L / 3 2 L / 32 L / 3 2 L / 32 L / 3
( P12CC ) FD = ∫ ∫ ρ12 ( x1 , x2 )dx1dx2 = ∫ ∫ ρ12 ( x1 , x2 )dx1dx2 − ∫ ∫ρ
FD classical int
12 ( x1 , x2 )dx1dx2
L/3 L/3 L/3 L/3 L/3 L/3

1 3  1 3
= 1
(P )(P ) + (P )(P ) − (I ) = (P )(P ) + (I )
C C 1 C C C 2 C C C 2
=  +  −



 3 2π  3 4π 
2 1 2 2 2 1 12 1 2 12

The 1st excited state has α = 1 and β = 3, Thus, E0FD = 10 E0 and

L/3 L/3 L/3 L/3 L/3 L/3


= ∫ ∫ρ ( x1 , x2 )dx1dx2 = ∫ ∫ρ ( x1 , x2 )dx1dx2 − ∫ ∫ρ
LL FD classical int
(P ) 13 FD 13 13 13 ( x1 , x2 )dx1dx2
0 0 0 0 0 0
2
1 3  1   3 3 
= P ( )(P ) − (I )
L L L 2
=  −   − 
 


 3 4π  3   8π 
1 3 13

Also,
L/3 L L/3 L L/3 L
( P13LR ) FD = ∫ ∫ ρ13 ( x1, x2 )dx1dx2 = ∫ ∫ ρ13 ( x1, x2 )dx1dx2 − ∫ ∫ρ
FD classical int
13 ( x1 , x2 )dx1dx2
0 2L / 3 0 2L / 3 0 2L / 3
2
1 3  1   3 3 
= 1
2
(P )(P ) + (P )(P ) − (I )(I ) = (P )(P ) − (I )(I )
1
L
3
R 1
2 3
L
1
R L
13
R
13 1
L
3
R L
13
R
13 =  −
3 4π
  − 
 3   8π 

   
Also,
2 L / 32 L / 3 2 L / 32 L / 3 2 L / 32 L / 3
= ∫ ∫ρ ( x1 , x2 )dx1dx2 = ∫ ∫ρ ( x1 , x2 )dx1dx2 − ∫ ∫ρ
CC FD classical int
(P ) 13 FD 13 13 13 ( x1 , x2 )dx1dx2
L/3 L/3 L/3 L/3 L/3 L/3
2
1 3  1   3 3 
= 1
(P )(P ) + (P )(P ) − (I ) = (P )(P ) − (I )
C C 1 C C C 2 C C C 2
=  +   − 
 


 3 2π  3   8π 
2 1 3 2 3 1 13 1 3 13

Department of Physics Page 25 of 25

You might also like